Monday, December 23, 2013

Weakening Conditional Reasoning


As proven by the discussion of the previous problem, there is a simple rule for
weakening a conditional conclusion:
To weaken a conditional conclusion, attack the necessary condition by
showing that the necessary condition does not need to occur in order for
the sufficient condition to occur.
This can be achieved by presenting a counterexample or by presenting
information that shows that the sufficient condition can occur without the
necessary condition.
This leads to another Classic Combination:
When you have conditional reasoning in the stimulus and a Weaken
question, immediately look for an answer that attacks the necessary
condition.
Please take a moment to complete the following problem:
4. Speaker: Contemporary business firms need to recognize that avoiding social responsibility
leads to the gradual erosion of power. This is Davis and Blomstrom’s Iron Law of
Responsibility: “In the long run, those who do not use power in a manner which society
considers responsible will tend to lose it.” The law’s application to human institutions
certainly stands confirmed by history. Though the “long run” may require decades or even
centuries in some instances, society ultimately acts to reduce power when society thinks it is
not being used responsibly. Therefore, a business that wishes to retain its power as long
as it can must act responsibly.
Which one of the following statements, if true, most
weakens the speaker’s argument?
(A) Government institutions are as subject to the
Iron Law of Responsibility as business
institutions.
(B) Public relations programs can cause society to
consider an institution socially responsible
even when it is not.
(C) The power of some institutions erodes more
slowly than the power of others, whether they
are socially responsible or not.
(D) Since no institution is eternal, every business
will eventually fail.
(E) Some businesses that have used power in
socially responsible ways have lost it.
This problem is similar in form to the carpet market problem. The conclusion
appears at the end and is conditional in nature: “a business that wishes to retain
its power as long as it can must act responsibly.”
This relationship can be diagrammed as:
WRP = business wish(es) to retain power as long as possible
AR = act responsibly
WRP-------------> AR
Hopefully, you identified this conclusion as conditional when you read the
stimulus. As you read the question stem, you should have immediately
prephrased an answer that would allow the sufficient condition to occur without
the necessary condition, namely that a business that wishes to retain power does
not necessarily have to act responsibly. Let us examine the answer choices with
this idea in mind:
Answer choice (A): Because this answer addresses government institutions, this
cannot hurt the conclusion, which is about businesses. If anything, this may
slightly support the argument. In the middle of the stimulus, the Speaker
mentions that “The law’s application to human institutions certainly stands
confirmed by history.” This answer affirms that statement by adding
governments to the named list of human institutions.
Answer choice (B): This is the correct answer. If a public relations program can
cause society to think an institution is socially responsible even when it is not,
then an institution that wishes to retain power could act irresponsibly and then
get a public relations firm to cover up the activities. In this way, the institution
could wish to retain power but not act responsibly. Since this scenario allows
the sufficient condition to occur without the necessary, this weakens the
argument.
Answer choice (C): Many students hold this answer choice as a Contender. The
answer is incorrect because the stimulus contemplates varying rates of power
retention, especially between socially responsible and non-socially responsible
institutions. If you read this answer thinking that the stimulus indicated socially
responsible institutions do not lose power if socially responsible, then you made
a quasi-Mistaken Reversal of the stimulus. There is never a presumption in the
argument that power can be held indefinitely. If there were, this answer would
be much more attractive.
Answer choice (D): The conclusion is clear in saying, “a business that wishes to
retain power as long as it can...” The italicized phrase allows for the idea that
businesses will eventually lose power and ultimately fail. Thus, this answer
does not hurt the argument.
Answer choice (E): This is another attractive answer, and one that lured in
many test takers. The answer states that even though some businesses acted
responsibly (AR), they did not retain power (RP). If this difference between
retaining power and wishing to retain power (WRP) is ignored, then this answer
can be seen as attacking the Mistaken Reversal of the conclusion. As you
learned from the discussion of answer choice (C) of the carpet market question,
attacking the Mistaken Reversal of the conclusion does not hurt the conclusion.
However, this answer is attractive because not only does it address elements of
the conclusion, it also appears as the final answer choice. A test taker who did
not like any of the earlier answers would find this answer quite attractive.
In the previous two problems, the only conditionality has been the conditional
statement in the conclusion. However, Weaken questions can contain
conditional statements throughout the stimulus. Consider the following problem:

5. Politician: All nations that place a high tax on income produce thereby a negative incentive for technological innovation, and all nations in which technological innovation is hampered inevitably fall behind in the international arms race. Those nations that, through historical accident or the foolishness of their political leadership, wind up in a strategically
disadvantageous position are destined to lose their voice in world affairs. So if a nation wants to maintain its value system and way of life, it must not allow its highest tax bracket to exceed 30 percent of income.
Each of the following, if true, weakens the
politician’s argument EXCEPT:
(A) The top level of taxation must reach 45 percent
before taxation begins to deter inventors and
industrialists from introducing new
technologies and industries.
(B) Making a great deal of money is an
insignificant factor in driving technological
innovation.
(C) Falling behind in the international arms race
does not necessarily lead to a strategically
less advantageous position.
(D) Those nations that lose influence in the world
community do not necessarily suffer from a
threat to their value system or way of life.
(E) Allowing one’s country to lose its
technological edge, especially as concerns
weaponry, would be foolish rather than
merely a historical accident.
This problem is a complete conditional argument containing conditional
premises and a conditional conclusion. Here is a breakdown of the argument:
HT = nations that place a high tax on income
NI = negative incentive for technological innovation
FB = fall behind in the international arms race; also, wind up in
a strategically disadvantageous position
LV = lose voice in world affairs
The first sentence contains two sufficient condition indicators (the word “all”)
and can be diagrammed as a chain:
HT-------------> NI-------------> FB
The next sentence paraphrases “fall behind in the international arms race” as
“wind up in a strategically disadvantageous position” and can be diagrammed
as:
FB------------> LV
Because the two statements have FB in common, a single long chain can be
created:
HT-----------> NI---------> FB-------------> LV
From our discussion of conditional reasoning we know that a chain of this
length contains many inferences. The conclusion, when paraphrased, tries to
make a contrapositive:
The phrase “nation wants maintain its value system and way of life” is a
very rough equivalent of “wind up in a strategically disadvantageous
position” and “lose a voice in world affairs.” The paraphrase is not a
perfect equivalent because the conclusion discuses values, and the
premises do not. For our purposes, we will symbolize this condition as:
FB
and
LV
The phrase “must not allow its highest tax bracket to exceed 30 percent
of income” is the equivalent of HT. Thus, the diagram for the
conclusion is:
FB
and HT30
LV
Thus, based on the chain of reasoning provided, we have a reasonable
conclusion, but not a perfect one because the paraphrase was not exact. The
question stem is a WeakenX, which means that four of the answers will weaken
the argument and the one correct answer will either have no effect on the
argument or will strengthen the argument.
Answer choice (A): This answer attacks the necessary condition of the
conclusion by showing that taxes could exceed 30% before problems occurred.
Answer choice (B): This answer attacks the first half of the first sentence, which
states that high taxes necessarily produce a negative incentive for technological
innovation. Because taxes lower an individual’s income, the higher the tax, the
greater the relative restriction on making money. Answer choice (B) shows that
higher taxes would not necessarily produce low innovation because innovators
do not care about the amount they earn.
Answer choice (C): This answer attacks the part of the argument that equates
“fall behind in the international arms race” as “wind up in a strategically
disadvantageous position.” If the two are not equated, then the chain of
premises breaks down.
Answer choice (D): Like (C), this attacks a portion of the argument where the
author equates terms. In this case, the paraphrase in the conclusion was not
exact, and this answer exploits that gap.
Answer choice (E): This is the correct answer. The answer does not hurt the
argument because the stimulus specifically states that “Those nations that,
through historical accident or the foolishness of their political leadership, wind
up in a strategically disadvantageous position...” So, the actual reason the nation
ends up in a disadvantageous position is not critical. It could be either
foolishness or historical accident. So, an answer that asserts that it is foolishness
and not historical accident has no effect on the argument.

No comments:

Post a Comment